Soal Dewasa Seri 1 dari 2 Flashcards

1
Q

Mannitol is a useful diuretic for patients with decompensated heart failure

A

F

How well did you know this?
1
Not at all
2
3
4
5
Perfectly
2
Q

Which of the following conditions is likely to precipitate symptomatic heart failure in patients with previously compensated left ventricular contractile dysfunction ?

A

All of the above

How well did you know this?
1
Not at all
2
3
4
5
Perfectly
3
Q

Digoxin therapy decreases mortality rates in patients with chronic heart failure

A

F

How well did you know this?
1
Not at all
2
3
4
5
Perfectly
4
Q

Pulmonary capillary wedge pressures of 13 to 17 mmHg are commonly responsible for pulmonary vascular redistribution and interstitial edema on the chest roentgenogram

A

F

How well did you know this?
1
Not at all
2
3
4
5
Perfectly
5
Q

Which of the following statements is false regarding coronary thrombosis in UAP and non STEMI?

A

Thrombosis reduces embolization and facilitates intervention

How well did you know this?
1
Not at all
2
3
4
5
Perfectly
6
Q

A 55 year old man with hyperlipidemia and obesity undergoes PCI with sirolimus eluting stenting to LAD to treat his non STEMI. The PCI is successful. His BP is not elevated. Post PCI echocardiogram reveals normal ventricular function and mild mitral regurgitation. His discharge medications should include:

A

Aspirin 81 mg daily, clopidogrel 75 mg daily, a statin and a beta blocker

How well did you know this?
1
Not at all
2
3
4
5
Perfectly
7
Q

A 72 year old woman in severe respiratory distress was taken to the ER by paramedics who intubated her in the field. According to her husband, she started experiencing chest pain 2 hours ago. She has a history of hypertension and hyperlipidemia only and has been healthy until she experienced chest pain 2 hours ago. According to the ER physician, her BP is 80/60 mmHg on levophed and her HR is 110 bpm. Her ECG shows 5 mm ST elevation in the anterior leads. The ER is located 3 hours from your hospital with cath lab. The ER physician is certified to place IABP. What should be done next?

A

Fibrinolytic therapy, IABP and emergency transfer

How well did you know this?
1
Not at all
2
3
4
5
Perfectly
8
Q

The patient above is transferred to your hospital. You are waiting for her in the cath lab. She still has ST elevation and she still on levophed with a BP of 80/60 and HR of 120 bpm. She undergo emergent catheterization and is found to have multivessel disease with chronically occluded long RCA stenosis, 80% type A lesion in a moderate size CX artery and 100% LAD lesion. The proximal LAD is filled with thrombus but is a PCI approachable lesion. What should be done next?

A

Refer to CABG

How well did you know this?
1
Not at all
2
3
4
5
Perfectly
9
Q

After primary PCI, which of the following findings is associated with the worst mortality?

A

TIMI 2 myocardial perfusion grade

How well did you know this?
1
Not at all
2
3
4
5
Perfectly
10
Q

Immediate beta blockers should not be used in which of the following situations?

A

Systolic blood pressure < 100 mmHg

How well did you know this?
1
Not at all
2
3
4
5
Perfectly
11
Q

A 72-year-old white female, previously well controlled on a once-daily combination pill containing atenolol 50 mg and hydrochlorothiazide 25 mg, presents with a rise in her blood pressure to 170/110. You add 5 mg of lisinopril, and her creatinine rises from 1.1 to 1.9. What do you suspect?

A

Atherosclerotic renal artery stenosis

How well did you know this?
1
Not at all
2
3
4
5
Perfectly
12
Q

Which of the following scenarios illustrates the most appropriate first-line selection of antihypertensive agent in the hypertensive patient described?

A

A 32- year-old with chronic migraines is prescribed metoprolol

How well did you know this?
1
Not at all
2
3
4
5
Perfectly
13
Q

A 47-year-old male with diabetes presents as a new patient to your clinic. He does not recall any abnormal blood pressure readings. You find his blood pressure to be 138/86 on two readings during this visit. You should

A

Provide lifestyle counseling and recheck blood pressure within a few months

How well did you know this?
1
Not at all
2
3
4
5
Perfectly
14
Q

In which of the following patients is the goal LDL cholesterol not less than 100 mg/dL?

A

A 63-year-old man with hypertension, tobacco use, and a low HDL

How well did you know this?
1
Not at all
2
3
4
5
Perfectly
15
Q

Which of the following statements is false with respect to STEMI versus NSTEMI?

A

The thrombus in a STEMI is more platelet rich than is the thrombus of a NSTEMI

How well did you know this?
1
Not at all
2
3
4
5
Perfectly
16
Q

A 60-year-old man presents with an acute anterior MI. He receives tPA in the emergency room one hour after onset of symptoms with rapid resolution of pain and ST elevations. He is admitted to the coronary care unit feeling well, pain free, and with stable blood pressure and pulse. Which intervention is least appropriate next?

A

Urgent cardiac catheterization with possible percutaneous coronary intervention

How well did you know this?
1
Not at all
2
3
4
5
Perfectly
17
Q

A 72-year-old woman presents to your office with dyspnea and peripheral edema. On examination, her BP is 180/70 mm Hg and her pulse is 100 bpm. She has elevated jugular venous pressure, peripheral edema of the ankles, and a fourth heart sound. All of the following would be reasonable to obtain in the near future except

A

24-hour Holter monitor

How well did you know this?
1
Not at all
2
3
4
5
Perfectly
18
Q

Each of the following is a component of the atherogenic “metabolic syndrome”, except

A

Serum LDL > 140 mg/dL

How well did you know this?
1
Not at all
2
3
4
5
Perfectly
19
Q

A 62-year-old man with chronic congestive heart failure (CHF) due to coronary artery disease has been on a stable medical regimen of lisinopril, aspirin, and furosemide for years. He has been extremely compliant with medications and office visits. He closely monitors his weight daily. In addition to his CHF, he has hypertension, diabetes, and gout. Last month, he suffered a gout flare-up that is being treated with indomethacin. Over the last three weeks he has noticed a gradual decline in exercise tolerance, a weight gain of 5 pounds, and worsening pedal edema. What is the most appropriate course of action?

A

Discontinue the indomethacin

How well did you know this?
1
Not at all
2
3
4
5
Perfectly
20
Q

A 75-year-old man is diagnosed with aortic stenosis. What is the most likely etiology of his valvular disorder?

A

Degenerative calcific valve

How well did you know this?
1
Not at all
2
3
4
5
Perfectly
21
Q

A 65-year-old man with a history of rheumatic fever as a child underwent a root canal four weeks ago. Two weeks ago, he presented to the emergency department with fever, myalgias, and fatigue. Admission creatinine was 1.8 with microscopic hematuria. An echocardiogram revealed mitral valve vegetation, and he was started on intravenous antibiotics for presumed endocarditis. Blood cultures subsequently grow Stetococcus viridans. He responds well, with resolution of his fever and return of his creatinine to normal. However, on the day of discharge, you are called urgently to his room because of sudden shortness of breath. His examination is notable for labored breathing, a blood pressure of 90/55, crackles in the bases of the lungs, and II/VI systolic murmur. An ECG is notable for sinus tachycardia at 110 bpm but no ischemic changes. What is the best course of action?

A

Urgent echocardiogram and surgical consultation

22
Q

A 31-year-old woman is 28 weeks pregnant with her first child. She is relatively healthy and has had an uneventful pregnancy. Her obstetrician asked her to call you because she has had increasing difficulty breathing at night. In fact she is now using three pillows and occasionally wakes up gasping for air. In your office she appears dyspneic when speaking in full sentences. Her blood pressure is 95/65 and her pulse is 118. Lung examination is notable for crackles at both bases. Cardiac exam reveals a loud S1, and an extra sound in diastole. ECG shows deep inverted P waves in V1 and a rightward QRS axis. She has 1+ pitting edema in the lower extremities. Which of the following is true?

A

The likely diagnosis is mitral stenosis, and she should respond to furosemide and β-blockers

23
Q

Hypertension resolves in almost all patients after discontinuation of oral contraceptives

A

F

24
Q

Thiazide diuretics can contribute to each of the following metabolic effects, except :

A

Hypouricemia

25
Q

Reversible hepatotoxicity develops in 10 percents of patients

A

F

26
Q

Which of the following statements regarding diabetes mellitus as a cardiovascular risk factors is correct ?

A

Lifestyle modifications significantly reduce the rate of diabetes development in at-risk individuals

27
Q

Ticlopidine and clopidogrel reversibly affect platelet aggregation

A

F

28
Q

Thrombolytic therapy increases the incidence of early post – MI pericarditis

A

F

29
Q

The intensity of the diastolic murmur is closely related to the severity of mitral stenosis

A

F

30
Q

The typical murmur is of low frequency and heard best with the bell of the stethoscope placed along the left sternal border

A

F

31
Q

A 53-year-old man with chronic mitral regurgitation underwent echocardiographic examination, which demonstrated a mildly enlarged LV diastolic chamber size, a normal end-systolic dimension (38 mm), and an LV ejection fraction of 60 percent. He is asymptomatic, and has not experienced unusual dyspnea on exertion, orthopnea, or lower-extremity edema. For this patient, each of the following statements is correct regarding the timing of mitral valve surgery, except :

A

Chronic administration of an ACE inhibitor would delay the need for surgery

32
Q

A 47-year-old male with diabetes presents as a new patient to your clinic. He does not recall any abnormal blood pressure readings. You find his blood pressure to be 138/86 on two readings during this visit. You should

A

Provide lifestyle counseling and recheck blood pressure within a few months

33
Q

A 47-year-old male with type 2 diabetes returns for follow-up. You find his blood pressure to be 138/72 in the left arm and 142/74 in the right arm. One month ago, his blood pressures were 136/68 in the left arm and 140/72 in the right, and on the visit before that the readings were 138/70 in the left arm and 142/68 in the right arm. He has attempted to modify his diet and exercise. Urinalysis shows microalbuminuria. You recommend

A

Start lisinopril 5 mg po every day

34
Q

The absence of pulmonary rales on examination excludes the presence of an elevated pulmonary capillary pressure

A

F

35
Q

When anticoagulant therapy is required during pregnancy, warfarin is less desireable than heparin. Select the reason :

A

Warfarin crosses placental barrier and may cause fetal abnormality

36
Q

Besides relieving the anxiety and pain of myocardial infarction, morphine has additional beneficial cardiovascular effect. Which of the following describe an important Cardiovascular effect of Morphine ?

A

Dilates vein, reducing venous return

37
Q

A patient with hypertension (HTN) has CHF, with an EF of less than 30%. Her serum creatinine is stable at 2 mg per dL, and she is currently on digoxin and furosemide. Her BP is 152/90 mm Hg. Which antihypertensive agent is indicated?

A

Captopril

38
Q

The mechanisms of benefit for HF of beta-blockers include all of the following except :

A

Increased ventricular pressure

39
Q

Which of the following methods can improve patient adherence to antihypertensive therapy?

A

all of the above

40
Q

Which of the following would have the least risk patients contemplating pregnancy

A

Severe MR fc I

41
Q

Primary cause of of diastolic heart failure include all of the following, except:

A

Dilated cardiomyopathy

42
Q

Rekomendasi terapi farmakologis untuk memperbaiki simtom dan/atau mengurangi iskemik pasien angina stabil berdasarkan “ESC guideline Stable Angina Pectoris 2006”, kecuali:

A

Jika intoleransi terhadap beta blocker atau kurang efikasi, dianjurkan monoterapi dengan Calcium channel blocker (CCB), nitrat masa kerja panjang atau nicorandil

43
Q

Indikasi potensial revaskularisasi, pada angina pektoris stabil, kecuali:

A

Pasien yang asma bronchiale, karena tidak bisa diberikan beta blocker

44
Q

Beta blocker dibawah ini di eliminasi sebagian besar melalui ginjal maka tidak dianjurkan pada pasien dengan gagal ginjal, kecuali

A

Metoprolol

45
Q

Pernyataan dibawah ini ditemukan pada kelainan defek septum atrial (DSA), kecuali

A

EKG DSA primum : interval PR memanjang dan sumbu QRS berdeviasi kekanan.

46
Q

Pada keadaan jantung yang mempunyai risiko untuk terjangkit endokarditis yang tinggi & moderat diperlukan antibiotik profilaksis. Keadaan tersebut adalah dibawah ini kecuali

A

Kardiomiopati hipertrofi

47
Q

Preload adalah:

A

tekanan aorta dikurangi tekanan vena

48
Q

Isovolumetric contraction time adalah:

A

waktu lamanya ventrikel melakukan ejeksi

49
Q

Perubahan yang terjadi pada bendungan paru adalah:

A

semua yang disebut diatas

50
Q

Saturasi O2 adalah :

A

Jumlah O2 yang diikat oleh darah dibanding dengan kemampuan darah dalam mengikat O2